Difference between revisions of "2006 AMC 8 Problems/Problem 1"

m (Solution)
m (Solution)
Line 6: Line 6:
 
== Solution ==
 
== Solution ==
 
The three prices round to <math> \textdollar 2 </math>, <math> \textdollar 5 </math>, and <math> \textdollar 10 </math>, which has a sum of <math> \boxed{\textbf{(D)}\ 17} </math>
 
The three prices round to <math> \textdollar 2 </math>, <math> \textdollar 5 </math>, and <math> \textdollar 10 </math>, which has a sum of <math> \boxed{\textbf{(D)}\ 17} </math>
 
                                                                           
 
Attention. Annoucement-"DID YA WANNNNNNNNNNNNNNNNNNNNNAAAAAAAAAAAAAAAAAAAAAAAAAAAAAAAAAAAAAAAAAAAAAAAAAAAA BUILDDDDDDDDDDDDDDD DA SNOWVANNNNNNNNNNNNNNNNNNNNNNNNNNNNNNNNNNNNN?" Thank you for your patience and consideration.
 
  
 
==See Also==
 
==See Also==
 
{{AMC8 box|year=2006|before=First <br />Question|num-a=2}}
 
{{AMC8 box|year=2006|before=First <br />Question|num-a=2}}
 
{{MAA Notice}}
 
{{MAA Notice}}

Revision as of 19:25, 1 September 2018

Problem

Mindy made three purchases for $\textdollar 1.98$ dollars, $\textdollar 5.04$ dollars, and $\textdollar 9.89$ dollars. What was her total, to the nearest dollar?

$\textbf{(A)}\ 10\qquad\textbf{(B)}\ 15\qquad\textbf{(C)}\ 16\qquad\textbf{(D)}\ 17\qquad\textbf{(E)}\ 18$

Solution

The three prices round to $\textdollar 2$, $\textdollar 5$, and $\textdollar 10$, which has a sum of $\boxed{\textbf{(D)}\ 17}$

See Also

2006 AMC 8 (ProblemsAnswer KeyResources)
Preceded by
First
Question
Followed by
Problem 2
1 2 3 4 5 6 7 8 9 10 11 12 13 14 15 16 17 18 19 20 21 22 23 24 25
All AJHSME/AMC 8 Problems and Solutions

The problems on this page are copyrighted by the Mathematical Association of America's American Mathematics Competitions. AMC logo.png